納什均衡

當收益相等時的子博弈完美納什均衡似乎有問題

  • April 16, 2022

我的問題來自這個問題:來自 Maths stackexchange 的https://math.stackexchange.com/questions/2132846/game-theory-subgame-perfect-nash-equilibrium-in-a-sequential-game-with-identica 。

根據答案中給出的解釋,我試圖找到另一個博弈的子博弈完美納什均衡(SPNE):

在此處輸入圖像描述

在這裡,有 2 個玩家,都必須在 A 和 B 之間進行選擇。玩家 1 先移動,然後玩家 2 移動。

讓我們將玩家的策略定義為 $ (p_1, p_2) $ 在哪裡 $ p_1 $ 和 $ p_2 $ 是玩家 1 和玩家 2 分別選擇 A 的機率。

在左下角的子博弈中,分析很簡單:玩家 2 會選擇 A,因為它給出了更高的收益。

在右下角,玩家 2 面臨相同的收益,所以任何 $ p_2 \in [0, 1] $ 是最優的。我們將可能性分為三種情況:

1. $ p_2 > 0.5 $ :這里為玩家 1 選擇 B 的預期收益是 $ 2p_2 + (1-p_2)4 < 3 $ ,選擇 A 的收益。因此玩家 1 將選擇 A。這給了我們 SPNE $ (p_1, p_2) = (1, 1) $ .

2. $ p_2 < 0.5 $ :這里為玩家 1 選擇 B 的預期收益是 $ 2p_2 + (1-p_2)4 > 3 $ ,選擇 A 的收益。因此參與者 1 將選擇 B。這給了我們一個均衡族: $ (0, p_2), p_2 \in [0, 0.5) $ .

3. $ p_2 = 0.5 $ :在這種情況下,玩家 1 選擇 B 的預期收益為 $ 2p_2 + (1-p_2)4 = 3 $ ,選擇 A 的收益。因此,任何 $ p_1 \in [0, 1] $ 是最優的。這給了我們另一個平衡族: $ (p_1, 0.5), p_1 \in [0, 1] $ .

這讓我想到了我的第一個問題:

問題 1:我對 SPNE 的描述是否正確?也就是說,上面提到的所有均衡實際上都是 SPNE 嗎?此外,還有其他我錯過的 SPNE 嗎?

如果“它們實際上都是 SPNE”的答案是肯定的,那麼我認為第 2 點的均衡族存在問題。條件是玩家 1 選擇 B,當然所有 $ p_2 \in [0, 1] $ 是最優的,包括所有 $ p_2 \in [0, 0.5) $ . 但是玩家 2 知道選擇任何這樣的 $ p_2 < 0.5 $ 將意味著玩家 1 選擇 B,而玩家 2 最終獲得收益 1。但如果她改為選擇 $ p_2 > 0.5 $ 她可以確定玩家 1 選擇了 A,然後玩家 2 最終的收益為 4,這比 1 更好。那為什麼玩家 2 會選擇 $ p_2 < 0.5 $ ?

這引出了我的第二個問題:

問題2:如果 $ (0, p_2), p_2 \in [0, 0.5) $ 不是 SPNE,我的分析出了什麼問題?在這種情況下,反向歸納會失敗還是我犯了錯誤?另一方面,如果這些實際上是 SPNE,那麼是否有任何對 SPNE 的改進,或者其他一些不受我反對的解決方案概念?我在哪裡可以了解它們?

您的分析原則上是正確的,但您的符號不正確。完美資訊博弈中玩家的(行為)策略必須為該玩家的每個決策節點指定動作的機率分佈。由於玩家 2 有兩個這樣的節點,一個左節點和一個右節點,每個節點有兩個可能的動作,她的策略可以寫成對 $ (p_2^L,p_2^R) $ 選擇的機率 $ A $ 分別位於左側和右側節點。然後將策略配置文件表示為 $ (p_1,(p_2^L,p_2^R)) $ . 三組 SPNE 由下式給出

  1. $ (1,(1,p_2^R)) $ 和 $ p_2^R>\frac12 $ ,
  2. $ (0,(1,p_2^R)) $ 和 $ p_2^R<\frac12 $ , 和
  3. $ (p_1,(1,\frac12)) $ 和 $ 0\le p_1\le 1 $ .

你的問題“為什麼玩家 2 會選擇 $ p_2^R<\frac12 $ ? ” 已經由您自己回答:因為以玩家 1 選擇 B 為條件,它是最優的。請注意,您聲稱“如果她選擇 $ p_2^R>\frac12 $ 她可以確定玩家 1 選擇了 $ A $ " 是不正確的。這只有在玩家 1 可以在決定之前觀察玩家 2 的選擇時才成立。

引用自:https://economics.stackexchange.com/questions/51115